LSAT and Law School Admissions Forum

Get expert LSAT preparation and law school admissions advice from PowerScore Test Preparation.

User avatar
 Dave Killoran
PowerScore Staff
  • PowerScore Staff
  • Posts: 5853
  • Joined: Mar 25, 2011
|
#85988
Complete Question Explanation
(The complete setup for this game can be found here: lsat/viewtopic.php?f=302&t=8539)

The correct answer choice is (D)

In this Justify question you must find the answer that produces just a single solution to the game. This is what you should seek in the correct answer:

     • As both the blocks in the game rotate, they must be addressed within the correct answer, and
     so you should expect to see answers that address F or H or both, and K or L or both.

     • J must also be addressed because it is sequentially linked to the KL block, and while that
     limits its range of placement, it does not determine where it is placed.

     • Because the super-sequence has multiple placement options within the game, the correct
     answer must directly anchor at least part of the sequence. The easiest way for this to occur is
     by removing variables from the triple-options that start and finish the game.

     • One last tip: answers that include fewer variables are less likely to be correct.

Answer choice (A) is incorrect because while this answer orders the variables in each block, it does not limit the placement of those blocks in any way.

Answer choice (B) is incorrect because while it produces a FHKL block, it does not limit the placement of that block. This answer also addresses only three variables, the fewest of any answer.

Answer choice (C) is incorrect because while it produces a HFLKJ block, it does not limit the placement of that block, and the block has no impact on G and S.

Answer choice (D) is the correct answer. This answer starts by forming a GFH block, which from the triple-option on the first space means that only S remains to be presented first. The remainder of the answer produces a KLJ block. The GFH block must then be presented second, third, and fourth, and the KLJ block must be presented fifth, sixth, and seventh. This results in a single solution:
Capture19.PNG
Answer choice (E) produces two blocks—HF and GKL—but it does not limit J or S in any manner.
You do not have the required permissions to view the files attached to this post.
 christinep
  • Posts: 2
  • Joined: Oct 02, 2011
|
#2041
Can someone please explain why C is incorrect? I thought if F went before L then H would go before F and K would go after L ( _ H F K L J _ ). Then since G cannot go first that leaves the set up to be ( S H F K L J G ). Is it not right to infer that since G cannot go first and only the first and last spots are available G must go last? I understand how the D answer is correct, I just don't understand how C is not correct. Please advise.
User avatar
 Dave Killoran
PowerScore Staff
  • PowerScore Staff
  • Posts: 5853
  • Joined: Mar 25, 2011
|
#2044
Hi Christine,

I'm going to assume you are referring to question #19, as that is the only question with (D) as a correct answer in this game.

First, answer choice (C) produces a block that appears as follows: H F L K J (you have L and K reversed in your explanation). As you noted, this block leaves G and S unplaced. However, you appear to have assumed that the block must occupy spaces 2-6, when it is also possible that it occupies spaces 3-7 ( S G H F L K J ). Thus, there are multiple possible orders under this answer choice, and therefore answer choice (C) is incorrect.

Does that make sense? Please let me know. Thanks!
 christinep
  • Posts: 2
  • Joined: Oct 02, 2011
|
#2052
Thanks for the help! I get it now.

Get the most out of your LSAT Prep Plus subscription.

Analyze and track your performance with our Testing and Analytics Package.